Flabbergasting!

Calculus Level 5

0 sin ( x 2 ) x e x 2 1 d x \large \int_{0}^{\infty}\dfrac{\sin(x^{2})x}{e^{x^{2}}-1} \, dx

If the integral above can be expressed as 1 A ( π B coth ( π C ) + D ) \dfrac1A ( \pi^B \coth(\pi^C) + D ) , where A , B , C , D A,B,C,D are all integers, find A + B + C + D A+B+C+D .


The answer is 5.

This section requires Javascript.
You are seeing this because something didn't load right. We suggest you, (a) try refreshing the page, (b) enabling javascript if it is disabled on your browser and, finally, (c) loading the non-javascript version of this page . We're sorry about the hassle.

3 solutions

First Last
Dec 10, 2016

0 sin ( x 2 ) x e x 2 1 d x = 1 2 0 sin x e x 1 d x = \displaystyle\int_{0}^{\infty}\dfrac{\sin(x^{2})x}{e^{x^{2}}-1}dx = \frac{1}{2}\int_{0}^{\infty}\frac{\sin{x}}{e^x-1}dx =

0 n = 1 x 2 n 1 ( 1 ) n + 1 ( 2 n 1 ) ! ( e x 1 ) = by expanding the power series for sin \displaystyle\int_{0}^{\infty}\sum_{n=1}^{\infty}\frac{x^{2n-1}(-1)^{n+1}}{(2n-1)!(e^x-1)}=\quad\text{by expanding the power series for sin}

n = 1 ζ ( 2 n ) Γ ( 2 n ) ( 1 ) n + 1 ( 2 n 1 ) ! d x = n = 1 ( 1 ) n + 1 ζ ( 2 n ) by the relation between zeta and gamma \displaystyle\sum_{n=1}^{\infty}\frac{\zeta(2n)\Gamma(2n)(-1)^{n+1}}{(2n-1)!}dx = \sum_{n=1}^{\infty}(-1)^{n+1}\zeta(2n)\quad\text{by the relation between zeta and gamma}

n = 0 ζ ( 2 n ) x 2 n = x π 2 cot ( x π ) \displaystyle\sum_{n=0}^{\infty}\zeta(2n)x^{2n} = \frac{-x\pi}{2}\cot{(x\pi)}

Using x = i and a little adjustment for the power of -1 and ζ ( 0 ) = 1 2 \displaystyle\zeta(0) = \frac{-1}{2} :

n = 1 ζ ( 2 n ) ( 1 ) n + 1 = i π 2 cot ( i π ) 1 2 = 1 2 ( π coth ( π ) 1 ) \displaystyle\sum_{n=1}^{\infty}\zeta(2n)(-1)^{n+1} = \frac{i\pi}{2}\cot{(i\pi)} - \frac{1}{2} = \frac{1}{2}(\pi \coth{(\pi)} -1)

Remembering the 1/2 from the u sub, f ( x ) = x coth x 1 \displaystyle\quad f(x) = x\coth{x}-1

Hi I'm inviting you to this .

Aditya Kumar - 4 years, 6 months ago

I = 0 x sin x 2 e x 2 1 d x = 1 / 2 0 sin u e u 1 d u ( Taking x 2 = u ) = 1 / 2 k 1 0 e k u sin u d u = 1 2 k 1 1 k 2 + 1 I=\int_0^\infty \frac{x\sin x^2}{e^{x^2}-1}dx\\=1/2\int_{0}^{\infty}\frac{\sin u}{e^u-1}du\quad (\mbox{Taking}\ x^2=u)\\ =1/2\sum_{k\ge 1}\int_{0}^{\infty}e^{-ku}\sin u du\\=\frac{1}{2}\sum_{k\ge 1}\frac{1}{k^2+1} Now, we use the infinite product formula for sin h x \sin hx to get sin h π x = π x k 1 ( 1 + x 2 k 2 ) ln sin h π x = ln ( π x ) + k 1 ln ( 1 + x 2 k 2 ) \sin h\pi x=\pi x\prod_{k\ge 1}\left(1+\frac{x^2}{k^2}\right)\\\implies \ln \sin h\pi x=\ln(\pi x)+\sum_{k\ge 1}\ln\left(1+\frac{x^2}{k^2}\right) Differentiating with respect to x x implies π cot h π x = 1 x + 2 x k 1 1 k 2 + x 2 1 2 k 1 1 k 2 + 1 = 1 4 ( π cot h π 1 ) \pi \cot h \pi x=\frac{1}{x}+2x\sum_{k\ge 1}\frac{1}{k^2+x^2}\\\implies \frac{1}{2}\sum_{k\ge 1}\frac{1}{k^2+1}=\frac{1}{4}\left(\pi \cot h \pi -1\right) which results in the answer 4 + 1 + 1 1 = 5 4+1+1-1=\boxed{5} .

Kunal Gupta
Dec 3, 2016

The Function f f is: f ( x ) = x coth ( x ) 1 f(x) = x\coth(x) -1 I'm wondering no one has presented a solution even though it has 7 solvers.I invented this and a similar integral and put it up on MSE. Although I solved this one on my own, but the other one is still unsolved and can be found here .( My mse username is kunal)
To begin with the solution, let's consider the following integral: I ( a ) = 0 sin ( x 2 ) x a e x 2 1 d x \large{ I(a) =\int_{0}^{\infty}\dfrac{\sin(x^2)x^a}{e^{x^2}-1}dx } Now, it can be expressed as: I ( a ) = 0 r = 1 sin ( x 2 ) x a e r x 2 d x \large{I(a) = \int_{0}^{\infty}\sum_{r=1}^{\infty}\sin(x^2)x^{a}e^{-rx^2}dx} Also, sin ( x 2 ) = Im [ e i x 2 ] \sin(x^2) = \text{Im}[e^{ix^{2}}] . Hence, I ( a ) = Im [ r = 1 0 x a e x 2 ( r i ) d x ] \large{I(a) =\text{Im}\left[\sum_{r=1}^{\infty}\int_{0}^{\infty}x^{a}e^{-x^{2}(r-i)}dx \right]} Note that the summation and the integral can be interchanged because of Fubini's Theorem ( It can be proved using Dominated Convergence Theorem).
Substituting x 2 ( r i ) = t x^{2}(r-i) = t I ( a ) = Im [ r = 0 0 t a 1 2 e t 2 ( r i ) a + 1 2 d x ] \large{ I(a) = \text{Im}\left[\sum_{r=0}^{\infty}\int_{0}^{\infty}\dfrac{t^{\frac{a-1}{2}}e^{-t}}{2(r-i)^{\frac{a+1}{2}}}dx \right]} BY the definition of Gamma Function, we can write it as I ( a ) = Im [ Γ ( a + 1 2 ) 2 r = 1 1 ( r i ) a + 1 2 ] \large{I(a) = \text{Im}\left[\dfrac{\Gamma \left(\dfrac{a+1}{2}\right)}{2}\sum_{r=1}^{\infty}\dfrac{1}{(r-i)^{\frac{a+1}{2}}}\right] } Writing ( r i ) = r 2 + 1 e i tan 1 ( 1 r ) \large{ (r-i) = \sqrt{r^{2}+1}e^{-i\tan^{-1}\left(\frac{1}{r}\right)}} Our integral now becomes, I ( a ) = Γ ( a + 1 2 ) 2 Im [ r = 1 e i a + 1 2 tan 1 ( 1 r ) ( r 2 + 1 ) a + 1 4 ] \large{ I(a) = \dfrac{\Gamma\left(\dfrac{a+1}{2}\right)}{2}\text{Im}\left[ \sum_{r=1}^{\infty}\dfrac{e^{i\frac{a+1}{2}\tan^{-1}\left(\frac{1}{r}\right)}}{(r^{2}+1)^{\frac{a+1}{4}}} \right] } Taking the Imaginary part, we get I ( a ) = Γ ( a + 1 2 ) 2 [ r = 1 sin ( a + 1 2 tan 1 ( 1 r ) ) ( r 2 + 1 ) a + 1 4 ] \large{I(a) = \dfrac{\Gamma\left(\dfrac{a+1}{2}\right)}{2}\left[ \sum_{r=1}^{\infty}\dfrac{\sin\left(\dfrac{a+1}{2}\tan^{-1}\left(\dfrac{1}{r}\right)\right)}{(r^{2}+1)^{\frac{a+1}{4}}} \right] } Plug a = 1 a=1 to get: I ( 1 ) = Γ ( 1 ) 2 [ r = 1 1 r 2 + 1 ] \large{ I(1) = \dfrac{\Gamma(1)}{2}\left[ \sum_{r=1}^{\infty}\dfrac{1}{r^{2}+1}\right] } The sum can be evaluated by various methods, including but not limited to Fourier Series, Digamma Functions etc. and comes out to be r = 1 1 r 2 + 1 = 1 2 ( π coth ( π ) 1 ) \large{ \sum_{r=1}^{\infty}\dfrac{1}{r^{2}+1} = \dfrac{1}{2}(\pi\coth(\pi)-1) } Making our required integral to be 0 sin ( x 2 ) x e x 2 1 d x = 1 4 ( π coth ( π ) 1 ) 0.538337023734 \large{ \int_{0}^{\infty}\dfrac{\sin(x^{2})x}{e^{x^{2}}-1}dx =\dfrac{1}{4}(\pi\coth(\pi)-1) \approx 0.538337023734 } verified numerically, which completes the solution.

but how to solve give me a clue ?

Kushal Bose - 4 years, 6 months ago

@Mark Hennings I was particularly able to solve this case of this generalized integral which I(my other username is kunal) put up on mse! Have you got any insight as the answers I got say that no closed form exists

Kunal Gupta - 4 years, 6 months ago

@Pi Han Goh Please see it and tell of any error( I couldn't get your report) @Kushal Bose Here's the solution

Kunal Gupta - 4 years, 6 months ago

Log in to reply

What is the function of your f f ? I got f ( x ) f(x) is a constant function, f ( x ) = 0.538337023734 4 f(x) = 0.538337023734 \cdot 4 .

Pi Han Goh - 4 years, 6 months ago

Log in to reply

as it's obvious, the function can be made up to be x coth ( x ) 1 x\coth(x)-1

Kunal Gupta - 4 years, 6 months ago

Log in to reply

@Kunal Gupta Why can't f ( x ) = e 2 π ( π 1 ) + ( π + 1 ) e 2 π 1 1 0 7 f ( e ) f ( e ) = 46369076 f(x) = \dfrac{e^{2\pi}(\pi -1)+(\pi +1)}{e^{2\pi}-1} \Rightarrow \left \lfloor 10^7 \cdot f(e) \cdot f(e) \right \rfloor = \boxed{46369076} ?

Pi Han Goh - 4 years, 6 months ago

Log in to reply

@Pi Han Goh the question reads f ( π ) f(\pi) not f ( x ) f(x) so that makes the function(kind of) variable wrt x . So i replaced π \pi with x x to make it a function

Kunal Gupta - 4 years, 6 months ago

Log in to reply

@Kunal Gupta Okay, why can't f ( x ) = e 2 π ( π 1 ) + ( π + 1 ) e 2 π 1 + x 2 x 2 1 0 7 f ( e ) f ( e ) = 46369076 f(x) = \dfrac{e^{2\pi}(\pi -1)+(\pi +1)}{e^{2\pi}-1} +x^2 - x^2 \Rightarrow \left \lfloor 10^7 \cdot f(e) \cdot f(e) \right \rfloor = \boxed{46369076} ?

Pi Han Goh - 4 years, 6 months ago

Log in to reply

@Pi Han Goh haha! I know we can make out infinite number of functions! maybe the problem is wrongly worded but i think you got the crux and are pulling my leg! Can you suggest some edits so that the answer remains same and the problem is legally \text{legally} correct?!

Kunal Gupta - 4 years, 6 months ago

Log in to reply

@Kunal Gupta How about just asking for the numerical value of this integral instead?

Pi Han Goh - 4 years, 6 months ago

0 pending reports

×

Problem Loading...

Note Loading...

Set Loading...